Exercise Sheet 5

41) Let \(s_{n,k}\) be the Stirling numbers of the first kind. Prove \(s_{n,2} = (n-1) H_{n-1}\) for \(n \geq 2\), where \(H_n = \sum_{k=1}^n \frac{1}{k}\).

By definition: \(s_{n,k} = s_{(n-1, k-1)} + (n-1) s_{(n-1,k)}\)

We proceed by induction on \(n\):

42) Let \(S_{n,k}\) be the Stirling numbers of the second kind, that is, the number of partitions of the set \(\{1,2, \dots, n\}\) into \(k\) (non-empty subsets). Set \(F_k(x) := \sum_{n \geq k} S_{n,k} x^n\). Prove that:

\[\begin{gather*} F_1(x) = \frac{x}{1-x} \end{gather*}\]

First, \(S_{(n,1)} = 1\) since there is only one way to partition a set into size \(1\) subsets. Thus:

\[\begin{gather*} F_1 (x) = \sum_{n \geq 1} x^n = x \sum_{n \geq 0} x^n = x \frac{1}{1-x} = \frac{x}{1-x} \end{gather*}\]

and

\[\begin{gather*} F_2(x) = \frac{x^2}{(1-x)(1-2x)} \end{gather*}\]

\[\begin{gather*} F_2(x) = \sum_{n \geq 2} S_{n,2} x^n = \sum_{n \geq 2} (2^{n-1} - 1) x^n = \sum_{n \geq 2} 2^{n-1} x^n - x^n = \end{gather*}\]

we apply an index shift

\[\begin{gather*} = \sum_{n \geq 1} 2^nx^{n+1} - x^{n+1} = (\sum_{n \geq 1} 2^n x^{n+1})^\dagger - (\sum_{n \geq 1} x^{n+1})^\heartsuit = \end{gather*}\]

\[\begin{gather*} = \frac{2x^2}{1-2x} - \frac{x^2}{1-x} = \frac{2x^2(1-x)}{(1-2x)(1-x)} - \frac{2x^2-2x^3-x^2+2x^3}{(1-2x)(1-x)} = \frac{x^2}{(1-x)(1-2x)}, \end{gather*}\]

where

\[\begin{gather*} \dagger = \sum_{n \geq 1} x^{n+1} = x^2 \sum_{n \geq 0} x^n = x^2 \frac{1}{1-x} = \frac{x^2}{1-x} \end{gather*}\]

and

\[\begin{gather*} \heartsuit = \sum_{n \geq 1} 2^n x^{n+1} = 2x^2 \sum_{n \geq 0} 2^n x^n = 2x^2 \frac{1}{1-2x} = \frac{2x^2}{1-2x} \end{gather*}\]

Furthermore, show that the functions \(F_k(x)\) satisfy the recurrence relation \(F_k(x) = \frac{x}{1-kx}F_{k-1}(x)\)

we start with the right hand side:

\[\begin{gather*} \frac{x}{1-kx}F_{k-1}(x) = \frac{x}{1-kx} \sum_{n \geq k-1} S_{n,k-1} x^n = \end{gather*}\]

we replace by the series for the generating function:

\[\begin{gather*} x \sum_{n \geq 0} k^n x^n \sum_{n \geq k-1} S_{n,k-1} x^n = \end{gather*}\]

we apply the cauchy product with \(a_i = S_{i,k-1}, b_j = k^j\)

\[\begin{gather*} x \sum_{n \geq k-1} \sum_{i=k-1}^n S_{i, k-1} k^{n-i} x^n = \end{gather*}\]

drag the x into the sum and use the lemma: \(S_{n+1,k+1} = \sum_{j=k}^n (k+1)^{n-j} S_{j,k}\)

\[\begin{gather*} \sum_{n\geq k-1} S_{n+1,k} x^{n+1}= \end{gather*}\]

we apply an index shift in the sum

\[\begin{gather*} \sum_{n \geq k} S_{n,k} x^n \end{gather*}\]

and solve the recurrence.

Starting with:

\[\begin{gather*} F_k(x) = \frac{x}{1-kx} F_{k-1}(x) \end{gather*}\]

we substitute repeatedly:

\[\begin{gather*} F_k(x) = \frac{x}{1-kx} \frac{x}{1-(k-1)x} F_{k-2}(x) \end{gather*}\]

we apply the substitution until the last term \(F_1\)

\[\begin{gather*} F_k(x) = \frac{x^{k-1}}{(1-kx)(1-(k-1)x) \cdots (1-2x)} F_1(x) \end{gather*}\]

we substitute the definition of \(F_1 = \frac{x}{1-x}\) and multiply it into the term.

\[\begin{gather*} F_k(x) = \frac{x^{k}}{(1-kx)(1-(k-1)x)\cdots (1-2x) (1-x)} \end{gather*}\]

43) Prove the following identity:

\[\begin{gather*} x^n = \sum_{k=0}^{n} S_{n,k} (x)_{k} \quad (n \geq 0) \end{gather*}\]

We proceed by induction:

\[\begin{gather*} x^1 = S_{1,0} (x)_0 + S_{1,1} (x)_1 = 0 + 1 \times x \end{gather*}\]

For any case \(n=1, i \in \mathbb{N}\):

\[\begin{gather*} x^n = \sum_{k=0}^n S_{n,k}(x)_k \end{gather*}\]

\[\begin{gather*} x^n = x \times x^{n-1} = \end{gather*}\]

Let \(i = n-1\), we apply the induction step for \(i+1=n\):

\[\begin{gather*} = x \sum_{k=0}^{n-1} S_{n-1,k} (x)_{k} = \end{gather*}\]

we start the sum at 1 since \(S_{n-1,0}\), thus the first summand does not affect the sum’s value

\[\begin{gather*} = \sum_{k=1}^{n-1}S_{n-1,k} (x)_k x = \end{gather*}\]

by \(x_{k+1} = x_k (x-k) = x \times x_k - kx_k\) we get \(x \times x_k = x_{k+1} + k x_k\) by transforming the equation, which we can apply as follows:

\[\begin{gather*} = \sum_{k=1}^{n-1} S_{n-1,k}( (x)_{k+1} + k(x)_k ) = \sum_{k=1}^{n-1} S_{n-1,k} (x)_{k+1} + S_{n-1,k} k(x)_k = \sum_{k=1}^{n-1} S_{n-1,k} (x)_{k+1} + \sum_{k=1}^{n-1} S_{n-1,k} k (x)_k = \end{gather*}\]

let \(k' = k+1\), we set \(k+1 = k'\):

\[\begin{gather*} = \sum_{k'=2}^{n} S_{n-1,k'-1} (x)_{k'} + \sum_{k=1}^{n-1} S_{n-1,k} k (x)_k = \end{gather*}\]

since \(S_{n-1,0} = S_{n-1,n} = 0\):

\[\begin{gather*} = \sum_{k=1}^{n} S_{n-1,k-1} (x)_{k} + \sum_{k=1}^{n} S_{n-1,k} k (x)_k = \sum_{k=1}^{n} S_{n-1,k-1} (x)_{k} + k S_{n-1,k} (x)_k = \sum_{k=1}^{n} (S_{n-1,k-1} + k S_{n-1,k}) (x)_k = \end{gather*}\]

this is the definition of the recurrence relation of the Stirling numbers of the second kind:

\[\begin{gather*} = \sum_{k=1}^{n} S_{n,k} (x)_k = \sum_{k=0}^{n} S_{n,k} (x)_k \end{gather*}\]

44) Let \(A,B\) be two finite sets with \(|A| = n\) and \(|B| = k\). How many injective mappings \(f: A \rightarrow B\) are there?

We assume \(k \geq n\) since otherwise no mapping is possible. For the first element in \(n\), there are \(k\) possibilities. For the second one, \(k-1\), etc… For the last element in \(n\) there are still \(k-(n-1)\) choices, thus we have \(k(k-1)\times \dots \times (k-n+1)\) mappings, this is the falling factorial \(k^{\underline{n}}\) which surmounts to

\[\begin{gather*} \frac{k!}{(k-n)!} \end{gather*}\]

Furthermore, show that the number of surjective mappings \(f: A \rightarrow B\) equals \(k!S_{n,k}\).

Assume \(n \geq k\), otherwise no such mapping is possible. We want all possible partitions of \(A\) into \(k\) non-empty subsets which are calculated by \(S(n,k)\). For each of these non-empty partitions of elements in \(A\) we select one item in \(B\) for the elements in the subset of \(A\) to map to. For the first partition we have \(k\) options from \(B\), \(k-1\) for the second, etc. and for the last element in \(B\), 1 option remains. Thus \(k!\) options per partitioning exist, i.e. \(k!S_{n,k}\).

46) Compute:

\[\begin{gather*} a_n = \sum_{k=0}^{n} k^2 \end{gather*}\]

using generating functions: Firstly, we know:

\[\begin{gather*} \sum_{n \geq 0} z^n = \frac{1}{1-z} \end{gather*}\]

we take the derivative, applying the quotient rule and the chain rule

\[\begin{gather*} \sum_{n \geq 0} n z^{n-1} = \frac{1'(1-z) - 1(1-z)'}{(1-z)^2} \end{gather*}\]

\[\begin{gather*} \sum_{n \geq 0} n z^{n-1} = \frac{0 - (-1)}{(1-z)^2} \end{gather*}\]

we multiply by \(z\) in order to preserve \(z^n\) on the left side

\[\begin{gather*} \sum_{n \geq 0} n z^{n} = \frac{z}{(1-z)^2} \end{gather*}\]

we take the derivative, again applying quotient and chain rule

\[\begin{gather*} \sum_{n \geq 0} n^2 z^{n-1} = \frac{z'(1-z)^2 - z ((1-z)^2)'}{(1-z)^4} \end{gather*}\]

\[\begin{gather*} \sum_{n \geq 0} n^2 z^{n-1} = \frac{(1-z)^2 +2z(1-z)}{(1-z)^4} \end{gather*}\]

\[\begin{gather*} \sum_{n \geq 0} n^2 z^{n-1} = \frac{1-z+2z}{(1-z)^3} \end{gather*}\]

\[\begin{gather*} \sum_{n \geq 0} n^2 z^{n-1} = \frac{1+z}{(1-z)^3} \end{gather*}\]

we multiply by \(z\)

\[\begin{gather*} \sum_{n \geq 0} n^2 z^{n} = \frac{z+z^2}{(1-z)^3} \end{gather*}\]

Now, let \(\tilde F(z)\) be the generating function for coefficient \(n^2\), of which we just calculated the generating function. We take the sum with coefficient \(1\) of which we know the GF (\(\frac{1}{1-z}\)) and apply the cauchy product:

\[\begin{gather*} \sum_{i \geq 0} i^2z^n \times \sum_{j \geq 0} z^n = \sum_{n \geq 0} \sum_{k=0}^{n} k^2 z^n \end{gather*}\]

Therefore:

\[\begin{gather*} \tilde F(z) \times \frac{1}{1-z} = F(z) \end{gather*}\]

we substitute the previously calculated GF of \(\tilde F(z)\)

\[\begin{gather*} F(z) = \frac{z+z^2}{(1-z)^3}\times \frac{1}{1-z} = \frac{z+z^2}{(1-z)^4} \end{gather*}\]

Now we want to compute \([z^n]F(z)\), i.e.

\[\begin{gather*} [z^n]\frac{z+z^2}{(1-z)^4} \end{gather*}\]

\[\begin{gather*} [z^n]\frac{z}{(1-z)^4} + [z^n]\frac{z^2}{(1-z)^4} \end{gather*}\]

we apply an index shift, to remove the \(z\) and \(z^2\) respectively

\[\begin{gather*} [z^{n-1}]\frac{1}{(1-z)^4} + [z^{n-2}]\frac{1}{(1-z)^4} \end{gather*}\]

\[\begin{gather*} [z^{n-1}] (1-z)^{-4} + [z^{n-2}] (1-z)^{-4} \end{gather*}\] we apply the generalised binomial theorem

\[\begin{gather*} [z^{n-1}] \sum_{n \geq 0} \binom{-4}{n}(-1)^{-4-n}+z^n + [z^{n-2}] \sum_{n \geq 0} \binom{-4}{n}(-1)^{-4-n}+z^n \end{gather*}\]

now for the coefficient extraction we put in the respective indices

\[\begin{gather*} \binom{-4}{n-1}(-1)^{-4-n-1} + \binom{-4}{n-2}(-1)^{-4-n-2} \end{gather*}\]

we apply some arithmetic equalities regarding the powers of \(-1\)

\[\begin{gather*} \binom{-4}{n-1}(-1)^{n-1} + \binom{-4}{n-2}(-1)^{n-2} \end{gather*}\]

when writing out the respective quotients, we can drag the factors \(-1\) into the quotients

\[\begin{gather*} \frac{(-4)(-5) \dots (-4-(n-2)) (-1)^{n-1}}{(n-1)! } + \frac{(-4)(-5) \dots (-4-(n-3)) (-1)^{n-2}}{(n-2)!} = \end{gather*}\]

\[\begin{gather*} = \frac{(4)(5) \dots (n - 2 + 4)}{(n-1)!} + \frac{(4)(5) \dots (n - 3 + 4)}{(n-2)!} = \frac{(4)(5) \dots (n + 2)}{(n-1)!} + \frac{(4)(5) \dots (n + 1)}{(n-2)!} = \end{gather*}\]

\[\begin{gather*} = \binom{n+2}{3} \binom{n+1}{3} = \frac{(n+2)(n+1)n + (n+1)n(n-1)}{3!} = \frac{(2n+1)(n+1)n}{3!} \end{gather*}\]

48) Prove the following identity:

\[\begin{gather*} \sum_{n \geq 0} \binom{2n}{n}z^n = \frac{1}{\sqrt{1-4z}}. \end{gather*}\]

We start with the right hand side:

\[\begin{gather*} \frac{1}{\sqrt{1-4z}} = (1-4z)^{-\frac{1}{2}} = \end{gather*}\]

by the generalised binomial theorem with \(x = 1\) and \(y=-4z\), since \(x\) is \(1\), only \(y\) is considered.

\[\begin{gather*} = \sum_{n=0}^{\infty} \binom{-\frac{1}{2}}{n} (-4z)^n = \sum_{n=0}^{\infty} \binom{-\frac{1}{2}}{n} (-4)^n z^n = \end{gather*}\]

again, by the theorem to handle \(-\frac{1}{2}\) in the upper index

\[\begin{gather*} = \sum_{n=0}^{\infty} \frac{-\frac{1}{2} * (-\frac{1}{2} - 1) \times \dots \times (-\frac{1}{2} - n +1)}{n!} (-4^n) z^n = \end{gather*}\]

we extract (-^n) from each factor

\[\begin{gather*} = \sum_{n=0}^{\infty} (-4)^n (-\frac{1}{2}^n) \frac{1 \times 3 \times (2n - 1)}{n!} z^n = \end{gather*}\]

all odds as factorials are \(!!\)

\[\begin{gather*} = \sum_{n=0}^{\infty} 2^n \frac{(2n-1)!!}{n!} z^n = \end{gather*}\]

since \((2n-1)!! = \frac{(2n)!}{2^n n!}\) by definition

\[\begin{gather*} \sum_{n=0}^{\infty} 2^n \frac{\frac{(2n)!}{2^n n!}}{\frac{n!}{1}} z^n = \sum_{n=0}^{\infty} 2^n \frac{(2n)!}{2^n n! n!} z^n = \sum_{n=0}^{\infty} \binom{2n}{n} z^n \end{gather*}\]

49) Compute

\[\begin{gather*} [z^n] \frac{2+5z}{\sqrt{1-8z}} \end{gather*}\]

where \([z^n]\sum_{n \geq 0} a_n z^n := a_n\) is the coefficient extraction operator.

\[\begin{gather*} \frac{2+5z}{\sqrt{1-8z}} = (2+5z^2) \frac{1}{\sqrt{1-8z}} = \end{gather*}\]

we apply the generalised binomial theorem for \((1-8z)^{-0.5}\)

\[\begin{gather*} = (2+5z^2) \sum_{k=0}^{\infty} \binom{-0.5}{k} (-8z)^k = (2+5z^2) \sum_{k\geq 0} \binom{-0.5}{k} (-1)^k 8^k z^k = \end{gather*}\]

\[\begin{gather*} = 2 \sum_{k\geq 0} \binom{-0.5}{k} (-1)^k 8^k z^k + 5z^2 \sum_{k\geq 0} \binom{-0.5}{k} (-1)^k 8^k z^k = \end{gather*}\]

\[\begin{gather*} = 2 \sum_{k\geq 0} \binom{-0.5}{k} (-1)^k 8^k z^k + 5 \sum_{k\geq 0} \binom{-0.5}{k} (-1)^k 8^k z^{k+2} = \end{gather*}\]

but we want \(z^k\) for coefficient extraction

\[\begin{gather*} = 2 \sum_{k\geq 0} \binom{-0.5}{k} (-1)^k 8^k z^k + 5 \sum_{k\geq 2} \binom{-0.5}{k-2} (-1)^{k-2} 8^{k-2} z^{k} = \end{gather*}\]

we want the same indices in both sums so we extract the first two elements on the left sum

\[\begin{gather*} = 2 + 8z + (2 \sum_{k\geq 2} \binom{-0.5}{k} (-1)^k 8^k z^k + 5 \sum_{k\geq 2} \binom{-0.5}{k-2} (-1)^{k-2} 8^{k-2} z^{k} = \end{gather*}\]

\[\begin{gather*} = 2 + 8z + \sum_{k\geq 2} \left(\binom{-0.5}{k} (-1)^k 8^k 2 \right) + \left( \binom{-0.5}{k-2} (-1)^{k-2} 8^{k-2} 5 \right) z^{k} = \end{gather*}\]

therefore (since \((-1)^k = (-1)^{k-2}\)):

\[\begin{gather*} [z^n]= \begin{cases} 2, & \text{if}\ k=0 \\ 8, & \text{if}\ k=1 \\ \left(\binom{-0.5}{k} (-1)^k 8^k 2 \right) + \left( \binom{-0.5}{k-2} (-1)^{k} 8^{k-2} 5 \right) & \text{if}\ k \geq 2 \end{cases} \end{gather*}\]

50) Solve the following recurrence using generating functions.

\(a_{n+1} = 3 a_n -2\), for \(n \geq 0, a_0 = 2\)

\[\begin{gather*} \sum_{n=0}^{\infty} a_{n+1} z^{n+1} = 3 \sum_{n=0}^{\infty} a_{n} z^{n+1} - 2 \sum_{n=0}^{\infty} z^{n+1} \Leftrightarrow \end{gather*}\]

we want \(z^n\) in all of our sums

\[\begin{gather*} \Leftrightarrow \sum_{n=1}^{\infty} a_{n} z^{n} = 3z \sum_{n=1}^{\infty} a_{n} z^{n} - 2z \sum_{n=0}^{\infty} z^{n} \Leftrightarrow \end{gather*}\]

we can transform the sums to their respective generating functions on the right side, the first sum is simple, for the second we need to consider that \(a_n = 1\) so the generating function is \(\frac{1}{1-z}\)

\[\begin{gather*} \Leftrightarrow (\sum_{n=1}^{\infty} a_{n} z^{n}) + a_0z^0 - a_0z^0 = 3z F(z) - 2 \frac{z}{1-z} \Leftrightarrow \end{gather*}\]

now we want each sum to start at \(0\), so we add the \(0\)th summand to the left sum

\[\begin{gather*} \Leftrightarrow (\sum_{n=1}^{\infty} a_{n} z^{n}) + a_0z^0 - a_0z^0 = 3z F(z) - 2 \frac{z}{1-z} \Leftrightarrow \end{gather*}\]

we can drag the \(0\)-th summand into the sum, to shift the index

\[\begin{gather*} \Leftrightarrow (\sum_{n=0}^{\infty} a_{n} z^{n}) - a_0z^0 = 3z F(z) - 2 \frac{z}{1-z} \Leftrightarrow \end{gather*}\] we again replace by the generating function

\[\begin{gather*} \Leftrightarrow F(z) - a_0 = 3z F(z) - 2 \frac{z}{1-z} \Leftrightarrow \end{gather*}\]

\[\begin{gather*} \Leftrightarrow F(z) - 3z F(z) = - 2 \frac{z}{1-z} + a_0 \end{gather*}\]

\[\begin{gather*} \Leftrightarrow F(z) - 3z F(z) = - \frac{2z}{(1-z)(1-3z)} + \frac{a_0}{1-3z} \Leftrightarrow \end{gather*}\]

we apply partial fraction decomposition

\[\begin{gather*} \Leftrightarrow F(z) = \frac{1}{1-z} - \frac{1}{1-3z} + \frac{a_0}{(1-3z)} \Leftrightarrow \end{gather*}\]

\[\begin{gather*} \Leftrightarrow F(z) = (a_0 - 1) \frac{1}{(1-3z)} + \frac{a_0}{(1-3z)} \Leftrightarrow \end{gather*}\]

\[\begin{gather*} \Leftrightarrow F(z) = (a_0 - 1) \sum_{n=0}^{\infty} 3^n z^n + \sum_{n=0}^{\infty} z^n \Leftrightarrow \end{gather*}\]

\[\begin{gather*} \Leftrightarrow F(z) = \sum_{n=0}^{\infty} (((a_0 - 1)3n)+1) z^n \end{gather*}\]